Positive work along path [closed]










1












$begingroup$


Consider I have a simple formula for the work along some path (in 1 dimension):



$$W~=~int_x_0^x_1vecFcdot dvecx.$$



If I now move from left to right ($x_1 > x_0$) along the axis (assuming the X-axis points right) it is obvious that (using unit vectors along the axis):



$vecF=Fveci$



$vecx=xveci => dvecx=dxveci$



$vecFdvecx = (Fveci)(dxveci)=Fdx$



Integration gives (assuming constant force along path for simplicity):



$int_x_0^x_1vecFdvecx = int_x_0^x_1Fdx = F int_x_0^x_1dx = F(x_1-x_0) > 0$



Consider now I move from right to left ($x_1 < x_0$) but the force is also reversed, then:



$vecF=-Fveci$



$vecx=-xveci => dvecx=-dxveci$



$vecFdvecx = (-Fveci)(-dxveci)=Fdx$



$int_x_0^x_1vecFdvecx = int_x_0^x_1Fdx = F int_x_0^x_1dx = F(x_1-x_0) < 0$



Obviously, there is some terrible mistake here in my reasoning... If we move from one point to another along the force direction the work must be positive.










share|cite|improve this question











$endgroup$



closed as off-topic by sammy gerbil, Jon Custer, Chair, Cosmas Zachos, Bill N Nov 19 '18 at 15:33


This question appears to be off-topic. The users who voted to close gave this specific reason:


  • "Homework-like questions should ask about a specific physics concept and show some effort to work through the problem. We want our questions to be useful to the broader community, and to future users. See our meta site for more guidance on how to edit your question to make it better" – sammy gerbil, Jon Custer, Chair, Cosmas Zachos, Bill N
If this question can be reworded to fit the rules in the help center, please edit the question.















  • $begingroup$
    I believe in the second integral the limits should switch places, shoudl they not? EDIT: Wait, hold on, i am reading it once more.
    $endgroup$
    – DakkVader
    Nov 11 '18 at 9:43











  • $begingroup$
    But this integral does include orientation, this it should always be from $x_0$ to $x_1$. I believe.
    $endgroup$
    – rk85
    Nov 11 '18 at 9:45










  • $begingroup$
    I caught that as well, i am rereading it. As you say, something could be wrong here, we just need to find it
    $endgroup$
    – DakkVader
    Nov 11 '18 at 9:46










  • $begingroup$
    The limits on the integral tell you the direction of motion. Don't put a sign on the $dx$.
    $endgroup$
    – Bill N
    Nov 19 '18 at 15:33















1












$begingroup$


Consider I have a simple formula for the work along some path (in 1 dimension):



$$W~=~int_x_0^x_1vecFcdot dvecx.$$



If I now move from left to right ($x_1 > x_0$) along the axis (assuming the X-axis points right) it is obvious that (using unit vectors along the axis):



$vecF=Fveci$



$vecx=xveci => dvecx=dxveci$



$vecFdvecx = (Fveci)(dxveci)=Fdx$



Integration gives (assuming constant force along path for simplicity):



$int_x_0^x_1vecFdvecx = int_x_0^x_1Fdx = F int_x_0^x_1dx = F(x_1-x_0) > 0$



Consider now I move from right to left ($x_1 < x_0$) but the force is also reversed, then:



$vecF=-Fveci$



$vecx=-xveci => dvecx=-dxveci$



$vecFdvecx = (-Fveci)(-dxveci)=Fdx$



$int_x_0^x_1vecFdvecx = int_x_0^x_1Fdx = F int_x_0^x_1dx = F(x_1-x_0) < 0$



Obviously, there is some terrible mistake here in my reasoning... If we move from one point to another along the force direction the work must be positive.










share|cite|improve this question











$endgroup$



closed as off-topic by sammy gerbil, Jon Custer, Chair, Cosmas Zachos, Bill N Nov 19 '18 at 15:33


This question appears to be off-topic. The users who voted to close gave this specific reason:


  • "Homework-like questions should ask about a specific physics concept and show some effort to work through the problem. We want our questions to be useful to the broader community, and to future users. See our meta site for more guidance on how to edit your question to make it better" – sammy gerbil, Jon Custer, Chair, Cosmas Zachos, Bill N
If this question can be reworded to fit the rules in the help center, please edit the question.















  • $begingroup$
    I believe in the second integral the limits should switch places, shoudl they not? EDIT: Wait, hold on, i am reading it once more.
    $endgroup$
    – DakkVader
    Nov 11 '18 at 9:43











  • $begingroup$
    But this integral does include orientation, this it should always be from $x_0$ to $x_1$. I believe.
    $endgroup$
    – rk85
    Nov 11 '18 at 9:45










  • $begingroup$
    I caught that as well, i am rereading it. As you say, something could be wrong here, we just need to find it
    $endgroup$
    – DakkVader
    Nov 11 '18 at 9:46










  • $begingroup$
    The limits on the integral tell you the direction of motion. Don't put a sign on the $dx$.
    $endgroup$
    – Bill N
    Nov 19 '18 at 15:33













1












1








1





$begingroup$


Consider I have a simple formula for the work along some path (in 1 dimension):



$$W~=~int_x_0^x_1vecFcdot dvecx.$$



If I now move from left to right ($x_1 > x_0$) along the axis (assuming the X-axis points right) it is obvious that (using unit vectors along the axis):



$vecF=Fveci$



$vecx=xveci => dvecx=dxveci$



$vecFdvecx = (Fveci)(dxveci)=Fdx$



Integration gives (assuming constant force along path for simplicity):



$int_x_0^x_1vecFdvecx = int_x_0^x_1Fdx = F int_x_0^x_1dx = F(x_1-x_0) > 0$



Consider now I move from right to left ($x_1 < x_0$) but the force is also reversed, then:



$vecF=-Fveci$



$vecx=-xveci => dvecx=-dxveci$



$vecFdvecx = (-Fveci)(-dxveci)=Fdx$



$int_x_0^x_1vecFdvecx = int_x_0^x_1Fdx = F int_x_0^x_1dx = F(x_1-x_0) < 0$



Obviously, there is some terrible mistake here in my reasoning... If we move from one point to another along the force direction the work must be positive.










share|cite|improve this question











$endgroup$




Consider I have a simple formula for the work along some path (in 1 dimension):



$$W~=~int_x_0^x_1vecFcdot dvecx.$$



If I now move from left to right ($x_1 > x_0$) along the axis (assuming the X-axis points right) it is obvious that (using unit vectors along the axis):



$vecF=Fveci$



$vecx=xveci => dvecx=dxveci$



$vecFdvecx = (Fveci)(dxveci)=Fdx$



Integration gives (assuming constant force along path for simplicity):



$int_x_0^x_1vecFdvecx = int_x_0^x_1Fdx = F int_x_0^x_1dx = F(x_1-x_0) > 0$



Consider now I move from right to left ($x_1 < x_0$) but the force is also reversed, then:



$vecF=-Fveci$



$vecx=-xveci => dvecx=-dxveci$



$vecFdvecx = (-Fveci)(-dxveci)=Fdx$



$int_x_0^x_1vecFdvecx = int_x_0^x_1Fdx = F int_x_0^x_1dx = F(x_1-x_0) < 0$



Obviously, there is some terrible mistake here in my reasoning... If we move from one point to another along the force direction the work must be positive.







newtonian-mechanics work vectors coordinate-systems conventions






share|cite|improve this question















share|cite|improve this question













share|cite|improve this question




share|cite|improve this question








edited Nov 11 '18 at 9:55









Qmechanic

103k121871183




103k121871183










asked Nov 11 '18 at 9:36









rk85rk85

371




371




closed as off-topic by sammy gerbil, Jon Custer, Chair, Cosmas Zachos, Bill N Nov 19 '18 at 15:33


This question appears to be off-topic. The users who voted to close gave this specific reason:


  • "Homework-like questions should ask about a specific physics concept and show some effort to work through the problem. We want our questions to be useful to the broader community, and to future users. See our meta site for more guidance on how to edit your question to make it better" – sammy gerbil, Jon Custer, Chair, Cosmas Zachos, Bill N
If this question can be reworded to fit the rules in the help center, please edit the question.







closed as off-topic by sammy gerbil, Jon Custer, Chair, Cosmas Zachos, Bill N Nov 19 '18 at 15:33


This question appears to be off-topic. The users who voted to close gave this specific reason:


  • "Homework-like questions should ask about a specific physics concept and show some effort to work through the problem. We want our questions to be useful to the broader community, and to future users. See our meta site for more guidance on how to edit your question to make it better" – sammy gerbil, Jon Custer, Chair, Cosmas Zachos, Bill N
If this question can be reworded to fit the rules in the help center, please edit the question.











  • $begingroup$
    I believe in the second integral the limits should switch places, shoudl they not? EDIT: Wait, hold on, i am reading it once more.
    $endgroup$
    – DakkVader
    Nov 11 '18 at 9:43











  • $begingroup$
    But this integral does include orientation, this it should always be from $x_0$ to $x_1$. I believe.
    $endgroup$
    – rk85
    Nov 11 '18 at 9:45










  • $begingroup$
    I caught that as well, i am rereading it. As you say, something could be wrong here, we just need to find it
    $endgroup$
    – DakkVader
    Nov 11 '18 at 9:46










  • $begingroup$
    The limits on the integral tell you the direction of motion. Don't put a sign on the $dx$.
    $endgroup$
    – Bill N
    Nov 19 '18 at 15:33
















  • $begingroup$
    I believe in the second integral the limits should switch places, shoudl they not? EDIT: Wait, hold on, i am reading it once more.
    $endgroup$
    – DakkVader
    Nov 11 '18 at 9:43











  • $begingroup$
    But this integral does include orientation, this it should always be from $x_0$ to $x_1$. I believe.
    $endgroup$
    – rk85
    Nov 11 '18 at 9:45










  • $begingroup$
    I caught that as well, i am rereading it. As you say, something could be wrong here, we just need to find it
    $endgroup$
    – DakkVader
    Nov 11 '18 at 9:46










  • $begingroup$
    The limits on the integral tell you the direction of motion. Don't put a sign on the $dx$.
    $endgroup$
    – Bill N
    Nov 19 '18 at 15:33















$begingroup$
I believe in the second integral the limits should switch places, shoudl they not? EDIT: Wait, hold on, i am reading it once more.
$endgroup$
– DakkVader
Nov 11 '18 at 9:43





$begingroup$
I believe in the second integral the limits should switch places, shoudl they not? EDIT: Wait, hold on, i am reading it once more.
$endgroup$
– DakkVader
Nov 11 '18 at 9:43













$begingroup$
But this integral does include orientation, this it should always be from $x_0$ to $x_1$. I believe.
$endgroup$
– rk85
Nov 11 '18 at 9:45




$begingroup$
But this integral does include orientation, this it should always be from $x_0$ to $x_1$. I believe.
$endgroup$
– rk85
Nov 11 '18 at 9:45












$begingroup$
I caught that as well, i am rereading it. As you say, something could be wrong here, we just need to find it
$endgroup$
– DakkVader
Nov 11 '18 at 9:46




$begingroup$
I caught that as well, i am rereading it. As you say, something could be wrong here, we just need to find it
$endgroup$
– DakkVader
Nov 11 '18 at 9:46












$begingroup$
The limits on the integral tell you the direction of motion. Don't put a sign on the $dx$.
$endgroup$
– Bill N
Nov 19 '18 at 15:33




$begingroup$
The limits on the integral tell you the direction of motion. Don't put a sign on the $dx$.
$endgroup$
– Bill N
Nov 19 '18 at 15:33










2 Answers
2






active

oldest

votes


















3












$begingroup$

Your mistake is in thinking that $dx$ doesn't account for the direction of motion. More explicitly, $$dboldsymbolx neq -dx hatboldsymboli$$ in the second scenario. Instead, $$dboldsymbolx = dx hatboldsymboli$$ as in the first case.






share|cite|improve this answer









$endgroup$












  • $begingroup$
    Thank you! I was looking for a stray - somewhere.
    $endgroup$
    – DakkVader
    Nov 11 '18 at 9:57










  • $begingroup$
    Thanks. But in the first case I move from left to right, in the second one, from right to left, thus the sign should change. Is that correct? In fact in seems it doesn't but why? Thanks in advance.
    $endgroup$
    – rk85
    Nov 11 '18 at 10:06










  • $begingroup$
    @rk85 It follows from the definition of a Riemann integral. Revisit it.
    $endgroup$
    – PiKindOfGuy
    Nov 11 '18 at 10:17






  • 1




    $begingroup$
    @rk85 PiKindOfGuy is correct. The quantity dx always points in the positive x direction. Just look at that quantity itself. When x increases it always increases in the positive x direction, regardless of the limits of integration. By flipping the sign you are doing a change of variable, which would require you to change your limits of integration in the normal fashion.
    $endgroup$
    – Dale
    Nov 11 '18 at 10:21


















0












$begingroup$

This answer amplifies @PiKindOfGuy's answer and @Dale's comment.



I think there is a confusion in the use and/or interpretation of
$dvec x$



  • as an infinitesimal element of a directed path
    and

  • as an increment of the x-coordinate.

Let's use $dvec s$ for the infinitesimal element of a path.

Let P and Q refer to start and end of the path.



So, $$W=int_P^Q vec Fcdot dvec s$$



Before I begin, I want to emphasize that the Force $vec F$ is generally independent of the path (and its direction) of the displacement. For instance, the gravitational force always points down... but the path could be arbitrary.



In your example, when you turn, you are changing the direction of the force and the displacement... let's say forward from P to Q then backward to R, where
$x_P ,x_R < x_Q$.



Along a horizontal displacement and constant magnitude force
...with no mention of any directions...
beginalign
W_AB
&=int_A^B vec Fcdot dvec s\
&=int_A^B (F_x hat i + F_y hat j) cdot (dx hat i)\
&=int_x_A^x_B F_x dx\
&=F_x (x_B-x_A)\
endalign



So, for the forward force $F_x=F$ along the forward path (displacement $(x_Q-x_P)>0$), we have
$$W_PQ= (F )(x_Q-x_P)>0.$$



For the backward force $F_x=-F$ along the backward path (displacement $(x_R-x_Q)<0$), we have
$$W_QR= (-F)(x_R-x_Q)>0.$$



Note that the lower limit and upper limit refer to the start and end of the oriented path... the limits control the orientation of the path.



Note the lower limit and the upper limits are not necessarily the smaller and larger of x-coordinates.






share|cite|improve this answer









$endgroup$



















    2 Answers
    2






    active

    oldest

    votes








    2 Answers
    2






    active

    oldest

    votes









    active

    oldest

    votes






    active

    oldest

    votes









    3












    $begingroup$

    Your mistake is in thinking that $dx$ doesn't account for the direction of motion. More explicitly, $$dboldsymbolx neq -dx hatboldsymboli$$ in the second scenario. Instead, $$dboldsymbolx = dx hatboldsymboli$$ as in the first case.






    share|cite|improve this answer









    $endgroup$












    • $begingroup$
      Thank you! I was looking for a stray - somewhere.
      $endgroup$
      – DakkVader
      Nov 11 '18 at 9:57










    • $begingroup$
      Thanks. But in the first case I move from left to right, in the second one, from right to left, thus the sign should change. Is that correct? In fact in seems it doesn't but why? Thanks in advance.
      $endgroup$
      – rk85
      Nov 11 '18 at 10:06










    • $begingroup$
      @rk85 It follows from the definition of a Riemann integral. Revisit it.
      $endgroup$
      – PiKindOfGuy
      Nov 11 '18 at 10:17






    • 1




      $begingroup$
      @rk85 PiKindOfGuy is correct. The quantity dx always points in the positive x direction. Just look at that quantity itself. When x increases it always increases in the positive x direction, regardless of the limits of integration. By flipping the sign you are doing a change of variable, which would require you to change your limits of integration in the normal fashion.
      $endgroup$
      – Dale
      Nov 11 '18 at 10:21















    3












    $begingroup$

    Your mistake is in thinking that $dx$ doesn't account for the direction of motion. More explicitly, $$dboldsymbolx neq -dx hatboldsymboli$$ in the second scenario. Instead, $$dboldsymbolx = dx hatboldsymboli$$ as in the first case.






    share|cite|improve this answer









    $endgroup$












    • $begingroup$
      Thank you! I was looking for a stray - somewhere.
      $endgroup$
      – DakkVader
      Nov 11 '18 at 9:57










    • $begingroup$
      Thanks. But in the first case I move from left to right, in the second one, from right to left, thus the sign should change. Is that correct? In fact in seems it doesn't but why? Thanks in advance.
      $endgroup$
      – rk85
      Nov 11 '18 at 10:06










    • $begingroup$
      @rk85 It follows from the definition of a Riemann integral. Revisit it.
      $endgroup$
      – PiKindOfGuy
      Nov 11 '18 at 10:17






    • 1




      $begingroup$
      @rk85 PiKindOfGuy is correct. The quantity dx always points in the positive x direction. Just look at that quantity itself. When x increases it always increases in the positive x direction, regardless of the limits of integration. By flipping the sign you are doing a change of variable, which would require you to change your limits of integration in the normal fashion.
      $endgroup$
      – Dale
      Nov 11 '18 at 10:21













    3












    3








    3





    $begingroup$

    Your mistake is in thinking that $dx$ doesn't account for the direction of motion. More explicitly, $$dboldsymbolx neq -dx hatboldsymboli$$ in the second scenario. Instead, $$dboldsymbolx = dx hatboldsymboli$$ as in the first case.






    share|cite|improve this answer









    $endgroup$



    Your mistake is in thinking that $dx$ doesn't account for the direction of motion. More explicitly, $$dboldsymbolx neq -dx hatboldsymboli$$ in the second scenario. Instead, $$dboldsymbolx = dx hatboldsymboli$$ as in the first case.







    share|cite|improve this answer












    share|cite|improve this answer



    share|cite|improve this answer










    answered Nov 11 '18 at 9:53









    PiKindOfGuyPiKindOfGuy

    482515




    482515











    • $begingroup$
      Thank you! I was looking for a stray - somewhere.
      $endgroup$
      – DakkVader
      Nov 11 '18 at 9:57










    • $begingroup$
      Thanks. But in the first case I move from left to right, in the second one, from right to left, thus the sign should change. Is that correct? In fact in seems it doesn't but why? Thanks in advance.
      $endgroup$
      – rk85
      Nov 11 '18 at 10:06










    • $begingroup$
      @rk85 It follows from the definition of a Riemann integral. Revisit it.
      $endgroup$
      – PiKindOfGuy
      Nov 11 '18 at 10:17






    • 1




      $begingroup$
      @rk85 PiKindOfGuy is correct. The quantity dx always points in the positive x direction. Just look at that quantity itself. When x increases it always increases in the positive x direction, regardless of the limits of integration. By flipping the sign you are doing a change of variable, which would require you to change your limits of integration in the normal fashion.
      $endgroup$
      – Dale
      Nov 11 '18 at 10:21
















    • $begingroup$
      Thank you! I was looking for a stray - somewhere.
      $endgroup$
      – DakkVader
      Nov 11 '18 at 9:57










    • $begingroup$
      Thanks. But in the first case I move from left to right, in the second one, from right to left, thus the sign should change. Is that correct? In fact in seems it doesn't but why? Thanks in advance.
      $endgroup$
      – rk85
      Nov 11 '18 at 10:06










    • $begingroup$
      @rk85 It follows from the definition of a Riemann integral. Revisit it.
      $endgroup$
      – PiKindOfGuy
      Nov 11 '18 at 10:17






    • 1




      $begingroup$
      @rk85 PiKindOfGuy is correct. The quantity dx always points in the positive x direction. Just look at that quantity itself. When x increases it always increases in the positive x direction, regardless of the limits of integration. By flipping the sign you are doing a change of variable, which would require you to change your limits of integration in the normal fashion.
      $endgroup$
      – Dale
      Nov 11 '18 at 10:21















    $begingroup$
    Thank you! I was looking for a stray - somewhere.
    $endgroup$
    – DakkVader
    Nov 11 '18 at 9:57




    $begingroup$
    Thank you! I was looking for a stray - somewhere.
    $endgroup$
    – DakkVader
    Nov 11 '18 at 9:57












    $begingroup$
    Thanks. But in the first case I move from left to right, in the second one, from right to left, thus the sign should change. Is that correct? In fact in seems it doesn't but why? Thanks in advance.
    $endgroup$
    – rk85
    Nov 11 '18 at 10:06




    $begingroup$
    Thanks. But in the first case I move from left to right, in the second one, from right to left, thus the sign should change. Is that correct? In fact in seems it doesn't but why? Thanks in advance.
    $endgroup$
    – rk85
    Nov 11 '18 at 10:06












    $begingroup$
    @rk85 It follows from the definition of a Riemann integral. Revisit it.
    $endgroup$
    – PiKindOfGuy
    Nov 11 '18 at 10:17




    $begingroup$
    @rk85 It follows from the definition of a Riemann integral. Revisit it.
    $endgroup$
    – PiKindOfGuy
    Nov 11 '18 at 10:17




    1




    1




    $begingroup$
    @rk85 PiKindOfGuy is correct. The quantity dx always points in the positive x direction. Just look at that quantity itself. When x increases it always increases in the positive x direction, regardless of the limits of integration. By flipping the sign you are doing a change of variable, which would require you to change your limits of integration in the normal fashion.
    $endgroup$
    – Dale
    Nov 11 '18 at 10:21




    $begingroup$
    @rk85 PiKindOfGuy is correct. The quantity dx always points in the positive x direction. Just look at that quantity itself. When x increases it always increases in the positive x direction, regardless of the limits of integration. By flipping the sign you are doing a change of variable, which would require you to change your limits of integration in the normal fashion.
    $endgroup$
    – Dale
    Nov 11 '18 at 10:21











    0












    $begingroup$

    This answer amplifies @PiKindOfGuy's answer and @Dale's comment.



    I think there is a confusion in the use and/or interpretation of
    $dvec x$



    • as an infinitesimal element of a directed path
      and

    • as an increment of the x-coordinate.

    Let's use $dvec s$ for the infinitesimal element of a path.

    Let P and Q refer to start and end of the path.



    So, $$W=int_P^Q vec Fcdot dvec s$$



    Before I begin, I want to emphasize that the Force $vec F$ is generally independent of the path (and its direction) of the displacement. For instance, the gravitational force always points down... but the path could be arbitrary.



    In your example, when you turn, you are changing the direction of the force and the displacement... let's say forward from P to Q then backward to R, where
    $x_P ,x_R < x_Q$.



    Along a horizontal displacement and constant magnitude force
    ...with no mention of any directions...
    beginalign
    W_AB
    &=int_A^B vec Fcdot dvec s\
    &=int_A^B (F_x hat i + F_y hat j) cdot (dx hat i)\
    &=int_x_A^x_B F_x dx\
    &=F_x (x_B-x_A)\
    endalign



    So, for the forward force $F_x=F$ along the forward path (displacement $(x_Q-x_P)>0$), we have
    $$W_PQ= (F )(x_Q-x_P)>0.$$



    For the backward force $F_x=-F$ along the backward path (displacement $(x_R-x_Q)<0$), we have
    $$W_QR= (-F)(x_R-x_Q)>0.$$



    Note that the lower limit and upper limit refer to the start and end of the oriented path... the limits control the orientation of the path.



    Note the lower limit and the upper limits are not necessarily the smaller and larger of x-coordinates.






    share|cite|improve this answer









    $endgroup$

















      0












      $begingroup$

      This answer amplifies @PiKindOfGuy's answer and @Dale's comment.



      I think there is a confusion in the use and/or interpretation of
      $dvec x$



      • as an infinitesimal element of a directed path
        and

      • as an increment of the x-coordinate.

      Let's use $dvec s$ for the infinitesimal element of a path.

      Let P and Q refer to start and end of the path.



      So, $$W=int_P^Q vec Fcdot dvec s$$



      Before I begin, I want to emphasize that the Force $vec F$ is generally independent of the path (and its direction) of the displacement. For instance, the gravitational force always points down... but the path could be arbitrary.



      In your example, when you turn, you are changing the direction of the force and the displacement... let's say forward from P to Q then backward to R, where
      $x_P ,x_R < x_Q$.



      Along a horizontal displacement and constant magnitude force
      ...with no mention of any directions...
      beginalign
      W_AB
      &=int_A^B vec Fcdot dvec s\
      &=int_A^B (F_x hat i + F_y hat j) cdot (dx hat i)\
      &=int_x_A^x_B F_x dx\
      &=F_x (x_B-x_A)\
      endalign



      So, for the forward force $F_x=F$ along the forward path (displacement $(x_Q-x_P)>0$), we have
      $$W_PQ= (F )(x_Q-x_P)>0.$$



      For the backward force $F_x=-F$ along the backward path (displacement $(x_R-x_Q)<0$), we have
      $$W_QR= (-F)(x_R-x_Q)>0.$$



      Note that the lower limit and upper limit refer to the start and end of the oriented path... the limits control the orientation of the path.



      Note the lower limit and the upper limits are not necessarily the smaller and larger of x-coordinates.






      share|cite|improve this answer









      $endgroup$















        0












        0








        0





        $begingroup$

        This answer amplifies @PiKindOfGuy's answer and @Dale's comment.



        I think there is a confusion in the use and/or interpretation of
        $dvec x$



        • as an infinitesimal element of a directed path
          and

        • as an increment of the x-coordinate.

        Let's use $dvec s$ for the infinitesimal element of a path.

        Let P and Q refer to start and end of the path.



        So, $$W=int_P^Q vec Fcdot dvec s$$



        Before I begin, I want to emphasize that the Force $vec F$ is generally independent of the path (and its direction) of the displacement. For instance, the gravitational force always points down... but the path could be arbitrary.



        In your example, when you turn, you are changing the direction of the force and the displacement... let's say forward from P to Q then backward to R, where
        $x_P ,x_R < x_Q$.



        Along a horizontal displacement and constant magnitude force
        ...with no mention of any directions...
        beginalign
        W_AB
        &=int_A^B vec Fcdot dvec s\
        &=int_A^B (F_x hat i + F_y hat j) cdot (dx hat i)\
        &=int_x_A^x_B F_x dx\
        &=F_x (x_B-x_A)\
        endalign



        So, for the forward force $F_x=F$ along the forward path (displacement $(x_Q-x_P)>0$), we have
        $$W_PQ= (F )(x_Q-x_P)>0.$$



        For the backward force $F_x=-F$ along the backward path (displacement $(x_R-x_Q)<0$), we have
        $$W_QR= (-F)(x_R-x_Q)>0.$$



        Note that the lower limit and upper limit refer to the start and end of the oriented path... the limits control the orientation of the path.



        Note the lower limit and the upper limits are not necessarily the smaller and larger of x-coordinates.






        share|cite|improve this answer









        $endgroup$



        This answer amplifies @PiKindOfGuy's answer and @Dale's comment.



        I think there is a confusion in the use and/or interpretation of
        $dvec x$



        • as an infinitesimal element of a directed path
          and

        • as an increment of the x-coordinate.

        Let's use $dvec s$ for the infinitesimal element of a path.

        Let P and Q refer to start and end of the path.



        So, $$W=int_P^Q vec Fcdot dvec s$$



        Before I begin, I want to emphasize that the Force $vec F$ is generally independent of the path (and its direction) of the displacement. For instance, the gravitational force always points down... but the path could be arbitrary.



        In your example, when you turn, you are changing the direction of the force and the displacement... let's say forward from P to Q then backward to R, where
        $x_P ,x_R < x_Q$.



        Along a horizontal displacement and constant magnitude force
        ...with no mention of any directions...
        beginalign
        W_AB
        &=int_A^B vec Fcdot dvec s\
        &=int_A^B (F_x hat i + F_y hat j) cdot (dx hat i)\
        &=int_x_A^x_B F_x dx\
        &=F_x (x_B-x_A)\
        endalign



        So, for the forward force $F_x=F$ along the forward path (displacement $(x_Q-x_P)>0$), we have
        $$W_PQ= (F )(x_Q-x_P)>0.$$



        For the backward force $F_x=-F$ along the backward path (displacement $(x_R-x_Q)<0$), we have
        $$W_QR= (-F)(x_R-x_Q)>0.$$



        Note that the lower limit and upper limit refer to the start and end of the oriented path... the limits control the orientation of the path.



        Note the lower limit and the upper limits are not necessarily the smaller and larger of x-coordinates.







        share|cite|improve this answer












        share|cite|improve this answer



        share|cite|improve this answer










        answered Nov 11 '18 at 19:02









        robphyrobphy

        1,922238




        1,922238













            Popular posts from this blog

            𛂒𛀶,𛀽𛀑𛂀𛃧𛂓𛀙𛃆𛃑𛃷𛂟𛁡𛀢𛀟𛁤𛂽𛁕𛁪𛂟𛂯,𛁞𛂧𛀴𛁄𛁠𛁼𛂿𛀤 𛂘,𛁺𛂾𛃭𛃭𛃵𛀺,𛂣𛃍𛂖𛃶 𛀸𛃀𛂖𛁶𛁏𛁚 𛂢𛂞 𛁰𛂆𛀔,𛁸𛀽𛁓𛃋𛂇𛃧𛀧𛃣𛂐𛃇,𛂂𛃻𛃲𛁬𛃞𛀧𛃃𛀅 𛂭𛁠𛁡𛃇𛀷𛃓𛁥,𛁙𛁘𛁞𛃸𛁸𛃣𛁜,𛂛,𛃿,𛁯𛂘𛂌𛃛𛁱𛃌𛂈𛂇 𛁊𛃲,𛀕𛃴𛀜 𛀶𛂆𛀶𛃟𛂉𛀣,𛂐𛁞𛁾 𛁷𛂑𛁳𛂯𛀬𛃅,𛃶𛁼

            Edmonton

            Crossroads (UK TV series)